Difference between revisions of "1993 AIME Problems/Problem 7"

(Solution)
Line 2: Line 2:
 
Three numbers, <math>a_1\,</math>, <math>a_2\,</math>, <math>a_3\,</math>, are drawn randomly and without replacement from the [[set]] <math>\{1, 2, 3, \dots, 1000\}\,</math>.  Three other numbers, <math>b_1\,</math>, <math>b_2\,</math>, <math>b_3\,</math>, are then drawn randomly and without replacement from the remaining set of 997 numbers.  Let <math>p\,</math> be the [[probability]] that, after a suitable rotation, a brick of dimensions <math>a_1 \times a_2 \times a_3\,</math> can be enclosed in a box of dimensions <math>b_1 \times b_2 \times b_3\,</math>, with the sides of the brick [[parallel]] to the sides of the box.  If <math>p\,</math> is written as a [[fraction]] in lowest terms, what is the sum of the numerator and denominator?
 
Three numbers, <math>a_1\,</math>, <math>a_2\,</math>, <math>a_3\,</math>, are drawn randomly and without replacement from the [[set]] <math>\{1, 2, 3, \dots, 1000\}\,</math>.  Three other numbers, <math>b_1\,</math>, <math>b_2\,</math>, <math>b_3\,</math>, are then drawn randomly and without replacement from the remaining set of 997 numbers.  Let <math>p\,</math> be the [[probability]] that, after a suitable rotation, a brick of dimensions <math>a_1 \times a_2 \times a_3\,</math> can be enclosed in a box of dimensions <math>b_1 \times b_2 \times b_3\,</math>, with the sides of the brick [[parallel]] to the sides of the box.  If <math>p\,</math> is written as a [[fraction]] in lowest terms, what is the sum of the numerator and denominator?
  
== Solution ==
+
== Solution 1 ==
 
Call the six numbers selected <math>x_1 > x_2 > x_3 > x_4 > x_5 > x_6</math>. Clearly, <math>x_1</math> must be a dimension of the box, and <math>x_6</math> must be a dimension of the brick.  
 
Call the six numbers selected <math>x_1 > x_2 > x_3 > x_4 > x_5 > x_6</math>. Clearly, <math>x_1</math> must be a dimension of the box, and <math>x_6</math> must be a dimension of the brick.  
  
Line 12: Line 12:
  
 
'''Note''' that the <math>1000</math> in the problem, is not used, and is cleverly bypassed in the solution, because we can call our six numbers <math>x_1,x_2,x_3,x_4,x_5,x_6</math> whether they may be <math>1,2,3,4,5,6</math> or <math>999,5,3,998,997,891</math>.
 
'''Note''' that the <math>1000</math> in the problem, is not used, and is cleverly bypassed in the solution, because we can call our six numbers <math>x_1,x_2,x_3,x_4,x_5,x_6</math> whether they may be <math>1,2,3,4,5,6</math> or <math>999,5,3,998,997,891</math>.
 +
 +
==Solution 2==
 +
 +
There's a <math>\frac{1}{2}</math> probability of one dimension being smaller than the other corresponding dimension. Therefore, the probability of being able to have a brick in a box is <math>\frac{1}{8}</math>. However, we can have brick A and box B or brick B and box A. We need to multiply by 2. Our desired answer is <math>\frac{1}{4}</math>
  
 
== See also ==
 
== See also ==

Revision as of 11:45, 14 November 2014

Problem

Three numbers, $a_1\,$, $a_2\,$, $a_3\,$, are drawn randomly and without replacement from the set $\{1, 2, 3, \dots, 1000\}\,$. Three other numbers, $b_1\,$, $b_2\,$, $b_3\,$, are then drawn randomly and without replacement from the remaining set of 997 numbers. Let $p\,$ be the probability that, after a suitable rotation, a brick of dimensions $a_1 \times a_2 \times a_3\,$ can be enclosed in a box of dimensions $b_1 \times b_2 \times b_3\,$, with the sides of the brick parallel to the sides of the box. If $p\,$ is written as a fraction in lowest terms, what is the sum of the numerator and denominator?

Solution 1

Call the six numbers selected $x_1 > x_2 > x_3 > x_4 > x_5 > x_6$. Clearly, $x_1$ must be a dimension of the box, and $x_6$ must be a dimension of the brick.

  • If $x_2$ is a dimension of the box, then any of the other three remaining dimensions will work as a dimension of the box. That gives us $3$ possibilities.
  • If $x_2$ is not a dimension of the box but $x_3$ is, then both remaining dimensions will work as a dimension of the box. That gives us $2$ possibilities.
  • If $x_4$ is a dimension of the box but $x_2,\ x_3$ aren’t, there are no possibilities (same for $x_5$).

The total number of arrangements is ${6\choose3} = 20$; therefore, $p = \frac{3 + 2}{20} = \frac{1}{4}$, and the answer is $1 + 4 = \boxed{005}$.

Note that the $1000$ in the problem, is not used, and is cleverly bypassed in the solution, because we can call our six numbers $x_1,x_2,x_3,x_4,x_5,x_6$ whether they may be $1,2,3,4,5,6$ or $999,5,3,998,997,891$.

Solution 2

There's a $\frac{1}{2}$ probability of one dimension being smaller than the other corresponding dimension. Therefore, the probability of being able to have a brick in a box is $\frac{1}{8}$. However, we can have brick A and box B or brick B and box A. We need to multiply by 2. Our desired answer is $\frac{1}{4}$

See also

1993 AIME (ProblemsAnswer KeyResources)
Preceded by
Problem 6
Followed by
Problem 8
1 2 3 4 5 6 7 8 9 10 11 12 13 14 15
All AIME Problems and Solutions

The problems on this page are copyrighted by the Mathematical Association of America's American Mathematics Competitions. AMC logo.png